PT11.S2.Q06 - cigarette smoking has been shown

SeriousbirdSeriousbird Alum Member
edited August 2016 in Logical Reasoning 1278 karma
This is a principle question.

I got this wrong in both drilling and blind review.

So I thought I am looking for something that will do the following:
connect the premise to the conclusion
SO, say something about how something that is a health hazard should be banned

My reasoning:

A) This is the one I picked in BR. I thought the phrase "should not be allowed" could be a referent to banned. It could be taking it a bit to the extreme, i.e. making that extra assumption, but to me this seemed like the strongest answer choice.
B) The argument is not about misleading claims, but rather if something is a health hazard it should be banned. This answer choice does not do that.
C) This is stretching the argument to an extreme. This is like saying advertisements for vitamins should include all side effects etc. This answer choice says all health hazards associated with promoted products should be included. The argument says if a product has a health hazard, it should be banned.
D) This answer choice is irrelevant. Conforming to regulations and standards is information that is extraneous and the argument did not address.
E) I thought this was wrong because of the word ban while this answer choice is discussing promoting a product. I guess it could be correct because if it is not a health hazard then it would be healthful. This could be the contrapositive, "if a product does not promote smoking then it is not a health hazard" Then you would take the extra leap and say if it does not promote smoking then it is a healthful product, and you would just ignore the health hazard part since that is no longer relevant.

I'm really confused. In my reasoning, I did not address the "promote smoking" part, maybe that's where I went wrong. I still think this could be a big leap of assumptions. If someone can tell me if my reasoning is valid/reasonable and explain answer choices A and E to me, I would be most appreciative!

Comments

  • runiggyrunruniggyrun Alum Inactive Sage Inactive ⭐
    2481 karma
    So, for a "principle" you need to think the same way you would for a SA question (that's why 7Sage calls them pseudoSA questions). We are looking for the principle that covers/justifies the whole gap (more than the gap is OK, less is not). Unlike for a SA question, where the resulting argument needs to be airtight, for a justifying principle/PseudoSA there can be slight "misalignments" as long as they don't involve the core of the issue (the thing that needs to be "justified").
    Answer A doesn't cover the whole gap because it says "show people doing things that endanger their health", whereas the argument talks about "promoting" in general.
    You can promote smoking in many, many ways that don't show people smoking, so that's a pretty big loophole that's not addressed by A. (Incidentally there are laws in parts of the world that prohibit on screen smoking but stop short of banning all advertisements for smoking).
    Answer E fills the gap and then some "advertisements should promote only healthful products" is definitely more than we need, because it might lead to a ban on advertising for cars and bank accounts, but it does bridge the gap completely.
    If smoking is unhealthy AND advertisements can only promote healthful things, then the government should ban ads promoting smoking.

    You might notice that there is a slight disconnect even in E (that's why it's a "principle that would justify" rather than a pure SA). There's a slight leap between "advertisements should only do A" and "government should ban ads that do /A". This is OK. The principle that "advertisements should only do A" would indeed justify the actions of a government that would choose to ban advertisers from doing /A.
  • nye8870nye8870 Alum
    1749 karma
    @runiggyrun said:
    You might notice that there is a slight disconnect even in E
    Good point. The Qstem just asks for a principle to "most support argument". It seems ultimately we end up missing a statement which would make this a "valid" argument, which is "If an entity should -not do- something (promote unhealthy practices) then the government should ban that something."
Sign In or Register to comment.